Równania Eulera-Lagrange’a: Różnice pomiędzy wersjami

[wersja przejrzana][wersja przejrzana]
Usunięta treść Dodana treść
Micha7a (dyskusja | edycje)
m →‎Dowód: drobne redakcyjne
lit.
Linia 3:
Dla [[funkcjonał]]u:
: <math>S = \int\limits_{t_1}^{t_2} L(x(t),x'(t),t) dt</math>
rozwiązaniem rówaniarównania Eulera-Lagrange'a:
: <math>\frac{d}{dt} \left (\frac{\partial L}{\partial x'}\right ) - \frac{\partial L}{\partial x} = 0</math>
są funkcje <math>x(t)</math>, dla których <math>S</math> jest stacjonarne. To znaczy, że dla niewielkich odchyleń <math>x(t)</math>, <math>S</math> zmienia się nieznacznie. Jest to warunkiem koniecznym, żeby <math>S</math> przyjmowało dla <math>x(t)</math> [[ekstremum]].